Last visit was: 25 Apr 2024, 10:54 It is currently 25 Apr 2024, 10:54

Close
GMAT Club Daily Prep
Thank you for using the timer - this advanced tool can estimate your performance and suggest more practice questions. We have subscribed you to Daily Prep Questions via email.

Customized
for You

we will pick new questions that match your level based on your Timer History

Track
Your Progress

every week, we’ll send you an estimated GMAT score based on your performance

Practice
Pays

we will pick new questions that match your level based on your Timer History
Not interested in getting valuable practice questions and articles delivered to your email? No problem, unsubscribe here.
Close
Request Expert Reply
Confirm Cancel
SORT BY:
Date
User avatar
Intern
Intern
Joined: 19 Mar 2015
Posts: 48
Own Kudos [?]: 42 [12]
Given Kudos: 11
Send PM
Most Helpful Reply
RC & DI Moderator
Joined: 02 Aug 2009
Status:Math and DI Expert
Posts: 11178
Own Kudos [?]: 31925 [5]
Given Kudos: 290
Send PM
General Discussion
Math Expert
Joined: 02 Sep 2009
Posts: 92914
Own Kudos [?]: 618976 [0]
Given Kudos: 81595
Send PM
Manhattan Prep Instructor
Joined: 04 Dec 2015
Posts: 935
Own Kudos [?]: 1541 [1]
Given Kudos: 115
GMAT 1: 790 Q51 V49
GRE 1: Q170 V170
Send PM
Re: Is |x - 1| < 1 ? (1) (x - 1)^2 >1 (2) x < 0 [#permalink]
1
Kudos
Expert Reply
gmatgrl wrote:
Is |x-1| < 1 ?

(1) (x-1)^2 >1
(2) x < 0

I don't know the official answer to this. But can someone help with the solution.


Even if you aren't sure how to do the algebra, this is a good DS problem for case testing. That's because the numbers involved are pretty simple, and there's only a single variable. It's always possible, when testing cases, that you'll miss something - but it's also the best way to prove that a statement is insufficient, and it's better than just guessing or giving up because the algebra is complex.

(1)

Test extremes here. Start with a large number: x = 1000 fits the statement, since (1000-1)^2 is much greater than 1. Then, answer the question. Is |1000-1| < 1? No. It's greater.

Next, think about what you'd have to achieve to get a different answer, in this case, a yes. You'd need a much smaller value of x. The smallest positive value of x that could possibly fit the statement would be something like 2.0001. But that also gives a 'no' answer, since |2.0001-1| is still greater than 1.

Try a negative value, as well. x = -0.5 works. But again, |-0.5-1| is greater than 1, so the answer is 'no'.

If you always get a 'no', the statement is sufficient.

(2)

Same situation - test a couple of negative values of x and notice that you always get a 'no', so it's sufficient. It's nice, but not always necessary, to logically reason out why you're always getting the same answer. But if the problem is tough and you're short on time, it's okay to just notice that every case seems to give the same result and decide that the statement is sufficient.
Math Revolution GMAT Instructor
Joined: 16 Aug 2015
Posts: 10161
Own Kudos [?]: 16598 [0]
Given Kudos: 4
GMAT 1: 760 Q51 V42
GPA: 3.82
Send PM
Re: Is |x - 1| < 1 ? (1) (x - 1)^2 >1 (2) x < 0 [#permalink]
Expert Reply
Forget conventional ways of solving math questions. In DS, Variable approach is the easiest and quickest way to find the answer without actually solving the problem. Remember equal number of variables and independent equations ensures a solution.

Is |x-1| < 1 ?

(1) (x-1)^2 >1
(2) x < 0


In the original condition, there is 1 variable(x), which should match with the number of equations. So you need 1 equation. For 1) 1 equation, for 2) 1 equation, which is likely to make D the answer. Q becomes -1<x-1<1?, 0<x<2? and for 1), x-1<-1, 1<x-1--> x<0, 2<x is derived, which is no and sufficient.
For 1), also x<0, which is no and sufficient.
Thus, the answer is D.


 For cases where we need 1 more equation, such as original conditions with“1 variable”, or “2 variables and 1 equation”, or “3 variables and 2 equations”, we have 1 equation each in both 1) and 2). Therefore, there is 59 % chance that D is the answer, while A or B has 38% chance and C or E has 3% chance. Since D is most likely to be the answer using 1) and 2) separately according to DS definition. Obviously there may be cases where the answer is A, B, C or E.
Manager
Manager
Joined: 02 Feb 2016
Posts: 75
Own Kudos [?]: 45 [0]
Given Kudos: 40
GMAT 1: 690 Q43 V41
Send PM
Re: Is |x - 1| < 1 ? (1) (x - 1)^2 >1 (2) x < 0 [#permalink]
I understand the question and the second statement but the interpretation of statement (1) is a bit of an annoyance. Can somebody help with that?
Math Expert
Joined: 02 Sep 2009
Posts: 92914
Own Kudos [?]: 618976 [0]
Given Kudos: 81595
Send PM
Re: Is |x - 1| < 1 ? (1) (x - 1)^2 >1 (2) x < 0 [#permalink]
Expert Reply
TheMastermind wrote:
I understand the question and the second statement but the interpretation of statement (1) is a bit of an annoyance. Can somebody help with that?


Is |x-1| < 1 ?

(1) (x-1)^2 >1. Since both sides of the inequality are non-negative, we can safely take the square root to get \(|x-1| > 1\) (recall that \(\sqrt{x^2}=|x|\)). Sufficient.

(2) x < 0. The question above can be rephrased as "is \(-1 < x - 1 < 1\)?". Add 1 to all three parts: "is \(0 < x < 2\)?". Thus, given statement (\(x < 0\)) gives a NO answer to the question. Sufficient.

Answer: D.

Hope it's clear.
Manager
Manager
Joined: 02 Feb 2016
Posts: 75
Own Kudos [?]: 45 [0]
Given Kudos: 40
GMAT 1: 690 Q43 V41
Send PM
Re: Is |x - 1| < 1 ? (1) (x - 1)^2 >1 (2) x < 0 [#permalink]
Bunuel wrote:
TheMastermind wrote:
I understand the question and the second statement but the interpretation of statement (1) is a bit of an annoyance. Can somebody help with that?


Is |x-1| < 1 ?

(1) (x-1)^2 >1. Since both sides of the inequality are non-negative, we can safely take the square root to get \(|x-1| > 1\) (recall that \(\sqrt{x^2}=|x|\)). Sufficient.

(2) x < 0. The question above can be rephrased as "is \(-1 < x - 1 < 1\)?". Add 1 to all three parts: "is \(0 < x < 2\)?". Thus, given statement (\(x < 0\)) gives a NO answer to the question. Sufficient.

Answer: D.

Hope it's clear.


Oh yes, didn't think of taking the square root in statement (1). That certainly makes things easier. What if I wanted to do it without taking the square root? Taking 1 one to the LHS making it (x-1)^2 - 1 > 0. How would the simplification look like in that case?
Math Expert
Joined: 02 Sep 2009
Posts: 92914
Own Kudos [?]: 618976 [1]
Given Kudos: 81595
Send PM
Re: Is |x - 1| < 1 ? (1) (x - 1)^2 >1 (2) x < 0 [#permalink]
1
Kudos
Expert Reply
TheMastermind wrote:
Bunuel wrote:
TheMastermind wrote:
I understand the question and the second statement but the interpretation of statement (1) is a bit of an annoyance. Can somebody help with that?


Is |x-1| < 1 ?

(1) (x-1)^2 >1. Since both sides of the inequality are non-negative, we can safely take the square root to get \(|x-1| > 1\) (recall that \(\sqrt{x^2}=|x|\)). Sufficient.

(2) x < 0. The question above can be rephrased as "is \(-1 < x - 1 < 1\)?". Add 1 to all three parts: "is \(0 < x < 2\)?". Thus, given statement (\(x < 0\)) gives a NO answer to the question. Sufficient.

Answer: D.

Hope it's clear.


Oh yes, didn't think of taking the square root in statement (1). That certainly makes things easier. What if I wanted to do it without taking the square root? Taking 1 one to the LHS making it (x-1)^2 - 1 > 0. How would the simplification look like in that case?


\((x-1)^2 >1\);

\(x^2 - 2x + 1 > 1\);

\(x^2 - 2x > 0\);

\(x(x - 2) > 0\);

The roots are 0 and 2. ">" sign indicates that the solution is to the left of the smaller root and to the right of the larger root. Thus x < 0 and x > 2.

9. Inequalities



For more check Ultimate GMAT Quantitative Megathread

Manager
Manager
Joined: 24 Mar 2015
Status:love the club...
Posts: 220
Own Kudos [?]: 112 [0]
Given Kudos: 527
Send PM
Re: Is |x - 1| < 1 ? (1) (x - 1)^2 >1 (2) x < 0 [#permalink]
MathRevolution wrote:
Forget conventional ways of solving math questions. In DS, Variable approach is the easiest and quickest way to find the answer without actually solving the problem. Remember equal number of variables and independent equations ensures a solution.

Is |x-1| < 1 ?

(1) (x-1)^2 >1
(2) x < 0


In the original condition, there is 1 variable(x), which should match with the number of equations. So you need 1 equation. For 1) 1 equation, for 2) 1 equation, which is likely to make D the answer. Q becomes -1<x-1<1?, 0<x<2? and for 1), x-1<-1, 1<x-1--> x<0, 2<x is derived, which is no and sufficient.
For 1), also x<0, which is no and sufficient.
Thus, the answer is D.


 For cases where we need 1 more equation, such as original conditions with“1 variable”, or “2 variables and 1 equation”, or “3 variables and 2 equations”, we have 1 equation each in both 1) and 2). Therefore, there is 59 % chance that D is the answer, while A or B has 38% chance and C or E has 3% chance. Since D is most likely to be the answer using 1) and 2) separately according to DS definition. Obviously there may be cases where the answer is A, B, C or E.



hi

For equation # 1.
(x-1)^2 >1

|x-1| > 1

so,
x is greater than 2
OR x is less than 0
please correct me if I am missing something ...

another approach, however, brings us to the scenario as under...

(x-1)^2 >1

x(x-2) > 0

so,
x is greater than 0
OR x is greater than 2
please correct me if I am missing something ...

So both approaches get us to the correct answer. Would you please, however, say if there is anything wrong with the second approach..? if yes or no, please say to me the correct approach to follow when dealing questions such as this one ..

thanks in advance ...
User avatar
Non-Human User
Joined: 09 Sep 2013
Posts: 32675
Own Kudos [?]: 822 [0]
Given Kudos: 0
Send PM
Re: Is |x - 1| < 1 ? (1) (x - 1)^2 >1 (2) x < 0 [#permalink]
Hello from the GMAT Club BumpBot!

Thanks to another GMAT Club member, I have just discovered this valuable topic, yet it had no discussion for over a year. I am now bumping it up - doing my job. I think you may find it valuable (esp those replies with Kudos).

Want to see all other topics I dig out? Follow me (click follow button on profile). You will receive a summary of all topics I bump in your profile area as well as via email.
GMAT Club Bot
Re: Is |x - 1| < 1 ? (1) (x - 1)^2 >1 (2) x < 0 [#permalink]
Moderator:
Math Expert
92914 posts

Powered by phpBB © phpBB Group | Emoji artwork provided by EmojiOne